Question

Given the production function Q=K2L2 and the price of capital and labor as r=4 and w=8,...

Given the production function Q=K2L2 and the price of capital and labor as r=4 and w=8, respectively, if the goal is Q=2500 find the level of capital (K) and labor (L) to minimize cost.

Homework Answers

Answer #1

Method used in order to find result

Here In order to find level of K and K we have to :
Minimize : Total Cost(TC) = wL + rK = 8L + 4K
subject to L2K2 = Q = 2500

For that we have used Legrange multiplier, differentiation and First order condition and find the value of L and K that will minimize cost.

Level of L and K that will minimize Total Cost is L = 5 and K = 10

Know the answer?
Your Answer:

Post as a guest

Your Name:

What's your source?

Earn Coins

Coins can be redeemed for fabulous gifts.

Not the answer you're looking for?
Ask your own homework help question
Similar Questions
Given production function: Q=L3/5K1/5. Where L is labor, K is capital, w is wage rate, and...
Given production function: Q=L3/5K1/5. Where L is labor, K is capital, w is wage rate, and r is rental rate. What kinds of returns to scale does your firm face? Find cost minimizing level of L and K, and long run cost function.
A firm’s production function is Q! = min(4L ,5K ). The price of labor is w...
A firm’s production function is Q! = min(4L ,5K ). The price of labor is w and the price of capital is r. a) Derive the demand function of labor and capital respectively. How does the demand of capital change with the price of capital? b) Derive the long-run total cost function. Write down the equation of the long-run expansion path. c) Suppose capital is fixed at K = 8 in the short run. Derive the short-run total cost function....
A firm produces a product with labor and capital. Its production function is described by Q...
A firm produces a product with labor and capital. Its production function is described by Q = min(L, K). Let w and r be the prices of labor and capital, respectively. a) Find the equation for the firm’s long-run total cost curve as a function of quantity Q and input prices, w and r. b) Find the solution to the firm’s short-run cost minimization problem when capital is fixed at a quantity of 5 units (i.e., K = 5). Derive...
Suppose a firm’s production function is given by Q = L1/2*K1/2. The Marginal Product of Labor...
Suppose a firm’s production function is given by Q = L1/2*K1/2. The Marginal Product of Labor and the Marginal Product of Capital are given by: MPL = (K^1/2)/2L^1/2 & MPK = (L^1/2)/2K^1/2) a) (12 points) If the price of labor is w = 48, and the price of capital is r = 12, how much labor and capital should the firm hire in order to minimize the cost of production if the firm wants to produce output Q = 10?...
Suppose a firm has a production function given by q = 3L + K. The firm...
Suppose a firm has a production function given by q = 3L + K. The firm can purchase labor, L at a price w = 24, and capital, K at a price of r = 5. What is the firm’s total cost function?
a firm produces a product with labor and capital as inputs. The production function is described...
a firm produces a product with labor and capital as inputs. The production function is described by Q=LK. the marginal products associated with this production function are MPL=K and MPK=L. let w=1 and r=1 be the prices of labor and capital, respectively a) find the equation for the firms long-run total cost curve curve as a function of quantity Q b) solve the firms short-run cost-minimization problem when capital is fixed at a quantity of 5 units (ie.,K=5). derive the...
1. Consider the production function q=K2L0.5   a) Find the cost minimizing quantities of K and L...
1. Consider the production function q=K2L0.5   a) Find the cost minimizing quantities of K and L for q = 100, r as the price of K and w as the price of L. b) Find the cost minimizing quantities of K and L for q = 1000, r as the price of K and w as the price of L. Explain whether or not the output expansion [change from part a) to part b)] is labor saving or capital saving.
The production function is q = (10KL)/(K+L) where L = labor, K= capital The cost function...
The production function is q = (10KL)/(K+L) where L = labor, K= capital The cost function is C = wL + vK where w = wages and v = cost of capital Assume K is fixed in the short run at K = 20 a.) Find the short run cost function. Find also the short run average and marginal costs. b.) The shut-down price is defined as the minimum of average variable cost. For this cost function, what is the...
Suppose you own a firm that producing shoes using both capital and labor. The production function...
Suppose you own a firm that producing shoes using both capital and labor. The production function is q=f(K, L)=0.5K2 L4 . In long run both capital (K) and labor (L) are variable. Price for each pair of shoes is $50 (p=50), the wage rate is 0.04 (w=0.04) and the rental price for capital is 1 (r=1). Given those output and input prices, what is the profit maximizing input level of K and L (K* & L* )?
The Pear Corp produces high end consumer electronics using labor L and capital K according to...
The Pear Corp produces high end consumer electronics using labor L and capital K according to production function Q = F (L,K) = (100)(L^1/4)(K^1/4). Let the price of a unit of labor be given by W and the price of a unit of capital is given by R. The output price is P = 1 which Pear Corp takes as given for any choice of output level Q. Both labor and capital are fully adjustable. Set input price W =...
ADVERTISEMENT
Need Online Homework Help?

Get Answers For Free
Most questions answered within 1 hours.

Ask a Question
ADVERTISEMENT